2012 AMC 8 Problems/Problem 15

Revision as of 09:51, 24 November 2012 by Bharatputra (talk | contribs) (Created page with "The smallest number greater than 2 that leaves a remainder of 2 when divided by 3, 4, 5, or 6 lies between what numbers? <math> \textbf{(A)}\hspace{.05in}40\text{ and }50\qquad\...")
(diff) ← Older revision | Latest revision (diff) | Newer revision → (diff)

The smallest number greater than 2 that leaves a remainder of 2 when divided by 3, 4, 5, or 6 lies between what numbers?

$\textbf{(A)}\hspace{.05in}40\text{ and }50\qquad\textbf{(B)}\hspace{.05in}51\text{ and }55\qquad\textbf{(C)}\hspace{.05in}56\text{ and }60\qquad\textbf{(D)}\hspace{.05in}\text{61 and 65}\qquad\textbf{(E)}\hspace{.05in}\text{66 and 99}$